Complex numbers and conjugates.










2















Given that $|z|=√3$, solve the equation $$2overlinez+frac3iz=sqrt15.$$




How to solve this question without a calculator?










share|cite|improve this question























  • Could you edit your question using MathJax? It's unclear what you're asking and where the division symbol should be.
    – Aleksa
    Nov 11 at 11:16











  • @Vittal Kamath, so what is the answer did you get?
    – Dhamnekar Winod
    Nov 11 at 12:12















2















Given that $|z|=√3$, solve the equation $$2overlinez+frac3iz=sqrt15.$$




How to solve this question without a calculator?










share|cite|improve this question























  • Could you edit your question using MathJax? It's unclear what you're asking and where the division symbol should be.
    – Aleksa
    Nov 11 at 11:16











  • @Vittal Kamath, so what is the answer did you get?
    – Dhamnekar Winod
    Nov 11 at 12:12













2












2








2








Given that $|z|=√3$, solve the equation $$2overlinez+frac3iz=sqrt15.$$




How to solve this question without a calculator?










share|cite|improve this question
















Given that $|z|=√3$, solve the equation $$2overlinez+frac3iz=sqrt15.$$




How to solve this question without a calculator?







complex-numbers






share|cite|improve this question















share|cite|improve this question













share|cite|improve this question




share|cite|improve this question








edited Nov 11 at 11:25









user10354138

7,3592824




7,3592824










asked Nov 11 at 11:04









Vittal Kamath

385




385











  • Could you edit your question using MathJax? It's unclear what you're asking and where the division symbol should be.
    – Aleksa
    Nov 11 at 11:16











  • @Vittal Kamath, so what is the answer did you get?
    – Dhamnekar Winod
    Nov 11 at 12:12
















  • Could you edit your question using MathJax? It's unclear what you're asking and where the division symbol should be.
    – Aleksa
    Nov 11 at 11:16











  • @Vittal Kamath, so what is the answer did you get?
    – Dhamnekar Winod
    Nov 11 at 12:12















Could you edit your question using MathJax? It's unclear what you're asking and where the division symbol should be.
– Aleksa
Nov 11 at 11:16





Could you edit your question using MathJax? It's unclear what you're asking and where the division symbol should be.
– Aleksa
Nov 11 at 11:16













@Vittal Kamath, so what is the answer did you get?
– Dhamnekar Winod
Nov 11 at 12:12




@Vittal Kamath, so what is the answer did you get?
– Dhamnekar Winod
Nov 11 at 12:12










2 Answers
2






active

oldest

votes


















4














HINT



Multiplying by $z$ we obtain



$$2bar z+frac3iz=sqrt15 implies 2bar zz+frac3izzfrac i i=sqrt15z$$



then recall that $bar z z=|z|^2$.






share|cite|improve this answer




















  • ,what are the next steps to arrive at final answer?
    – Dhamnekar Winod
    Nov 11 at 12:13











  • @DhamnekarWinod What did you obtain from here $2bar zz+frac3izzfrac i i=sqrt15z$?
    – gimusi
    Nov 11 at 12:14










  • ,I got $6+ frac3i=sqrt45$
    – Dhamnekar Winod
    Nov 11 at 12:16











  • @DhamnekarWinod Why that? We have $bar z z=|z|^2$ and here we are ok, then $frac 3 i =-3i$ but at the RHS we should have $sqrt15z$.
    – gimusi
    Nov 11 at 12:21










  • because|z|=$sqrt3$. If this is wrong,then $z=frac6-3isqrt15$
    – Dhamnekar Winod
    Nov 11 at 12:31



















1














WLOG $z=sqrt3e^itimpliesbar z=sqrt3e^-it$ where $t$ is real



$$sqrt15=2sqrt3e^-it+dfrac3isqrt3e^it=sqrt3(2-i)e^-it$$



$$iff e^it=dfrac2-isqrt5$$



We are done.



We can go even further.



$$e^it=e^-iarcsindfrac1sqrt5$$



$$implies t=2npi -arcsindfrac1sqrt5$$ where $n$ is any integer






share|cite|improve this answer




















  • ,what is WLOG z means?
    – Dhamnekar Winod
    Nov 11 at 13:00










  • artofproblemsolving.com/wiki/…
    – lab bhattacharjee
    Nov 11 at 13:03










  • I know $e^ipi=-1$. So what is $e^it?$ and why did you multiply it by z?
    – Dhamnekar Winod
    Nov 11 at 13:11











  • math.stackexchange.com/questions/2660361/…
    – lab bhattacharjee
    Nov 11 at 13:12










Your Answer





StackExchange.ifUsing("editor", function ()
return StackExchange.using("mathjaxEditing", function ()
StackExchange.MarkdownEditor.creationCallbacks.add(function (editor, postfix)
StackExchange.mathjaxEditing.prepareWmdForMathJax(editor, postfix, [["$", "$"], ["\\(","\\)"]]);
);
);
, "mathjax-editing");

StackExchange.ready(function()
var channelOptions =
tags: "".split(" "),
id: "69"
;
initTagRenderer("".split(" "), "".split(" "), channelOptions);

StackExchange.using("externalEditor", function()
// Have to fire editor after snippets, if snippets enabled
if (StackExchange.settings.snippets.snippetsEnabled)
StackExchange.using("snippets", function()
createEditor();
);

else
createEditor();

);

function createEditor()
StackExchange.prepareEditor(
heartbeatType: 'answer',
autoActivateHeartbeat: false,
convertImagesToLinks: true,
noModals: true,
showLowRepImageUploadWarning: true,
reputationToPostImages: 10,
bindNavPrevention: true,
postfix: "",
imageUploader:
brandingHtml: "Powered by u003ca class="icon-imgur-white" href="https://imgur.com/"u003eu003c/au003e",
contentPolicyHtml: "User contributions licensed under u003ca href="https://creativecommons.org/licenses/by-sa/3.0/"u003ecc by-sa 3.0 with attribution requiredu003c/au003e u003ca href="https://stackoverflow.com/legal/content-policy"u003e(content policy)u003c/au003e",
allowUrls: true
,
noCode: true, onDemand: true,
discardSelector: ".discard-answer"
,immediatelyShowMarkdownHelp:true
);



);













draft saved

draft discarded


















StackExchange.ready(
function ()
StackExchange.openid.initPostLogin('.new-post-login', 'https%3a%2f%2fmath.stackexchange.com%2fquestions%2f2993720%2fcomplex-numbers-and-conjugates%23new-answer', 'question_page');

);

Post as a guest















Required, but never shown

























2 Answers
2






active

oldest

votes








2 Answers
2






active

oldest

votes









active

oldest

votes






active

oldest

votes









4














HINT



Multiplying by $z$ we obtain



$$2bar z+frac3iz=sqrt15 implies 2bar zz+frac3izzfrac i i=sqrt15z$$



then recall that $bar z z=|z|^2$.






share|cite|improve this answer




















  • ,what are the next steps to arrive at final answer?
    – Dhamnekar Winod
    Nov 11 at 12:13











  • @DhamnekarWinod What did you obtain from here $2bar zz+frac3izzfrac i i=sqrt15z$?
    – gimusi
    Nov 11 at 12:14










  • ,I got $6+ frac3i=sqrt45$
    – Dhamnekar Winod
    Nov 11 at 12:16











  • @DhamnekarWinod Why that? We have $bar z z=|z|^2$ and here we are ok, then $frac 3 i =-3i$ but at the RHS we should have $sqrt15z$.
    – gimusi
    Nov 11 at 12:21










  • because|z|=$sqrt3$. If this is wrong,then $z=frac6-3isqrt15$
    – Dhamnekar Winod
    Nov 11 at 12:31
















4














HINT



Multiplying by $z$ we obtain



$$2bar z+frac3iz=sqrt15 implies 2bar zz+frac3izzfrac i i=sqrt15z$$



then recall that $bar z z=|z|^2$.






share|cite|improve this answer




















  • ,what are the next steps to arrive at final answer?
    – Dhamnekar Winod
    Nov 11 at 12:13











  • @DhamnekarWinod What did you obtain from here $2bar zz+frac3izzfrac i i=sqrt15z$?
    – gimusi
    Nov 11 at 12:14










  • ,I got $6+ frac3i=sqrt45$
    – Dhamnekar Winod
    Nov 11 at 12:16











  • @DhamnekarWinod Why that? We have $bar z z=|z|^2$ and here we are ok, then $frac 3 i =-3i$ but at the RHS we should have $sqrt15z$.
    – gimusi
    Nov 11 at 12:21










  • because|z|=$sqrt3$. If this is wrong,then $z=frac6-3isqrt15$
    – Dhamnekar Winod
    Nov 11 at 12:31














4












4








4






HINT



Multiplying by $z$ we obtain



$$2bar z+frac3iz=sqrt15 implies 2bar zz+frac3izzfrac i i=sqrt15z$$



then recall that $bar z z=|z|^2$.






share|cite|improve this answer












HINT



Multiplying by $z$ we obtain



$$2bar z+frac3iz=sqrt15 implies 2bar zz+frac3izzfrac i i=sqrt15z$$



then recall that $bar z z=|z|^2$.







share|cite|improve this answer












share|cite|improve this answer



share|cite|improve this answer










answered Nov 11 at 11:16









gimusi

1




1











  • ,what are the next steps to arrive at final answer?
    – Dhamnekar Winod
    Nov 11 at 12:13











  • @DhamnekarWinod What did you obtain from here $2bar zz+frac3izzfrac i i=sqrt15z$?
    – gimusi
    Nov 11 at 12:14










  • ,I got $6+ frac3i=sqrt45$
    – Dhamnekar Winod
    Nov 11 at 12:16











  • @DhamnekarWinod Why that? We have $bar z z=|z|^2$ and here we are ok, then $frac 3 i =-3i$ but at the RHS we should have $sqrt15z$.
    – gimusi
    Nov 11 at 12:21










  • because|z|=$sqrt3$. If this is wrong,then $z=frac6-3isqrt15$
    – Dhamnekar Winod
    Nov 11 at 12:31

















  • ,what are the next steps to arrive at final answer?
    – Dhamnekar Winod
    Nov 11 at 12:13











  • @DhamnekarWinod What did you obtain from here $2bar zz+frac3izzfrac i i=sqrt15z$?
    – gimusi
    Nov 11 at 12:14










  • ,I got $6+ frac3i=sqrt45$
    – Dhamnekar Winod
    Nov 11 at 12:16











  • @DhamnekarWinod Why that? We have $bar z z=|z|^2$ and here we are ok, then $frac 3 i =-3i$ but at the RHS we should have $sqrt15z$.
    – gimusi
    Nov 11 at 12:21










  • because|z|=$sqrt3$. If this is wrong,then $z=frac6-3isqrt15$
    – Dhamnekar Winod
    Nov 11 at 12:31
















,what are the next steps to arrive at final answer?
– Dhamnekar Winod
Nov 11 at 12:13





,what are the next steps to arrive at final answer?
– Dhamnekar Winod
Nov 11 at 12:13













@DhamnekarWinod What did you obtain from here $2bar zz+frac3izzfrac i i=sqrt15z$?
– gimusi
Nov 11 at 12:14




@DhamnekarWinod What did you obtain from here $2bar zz+frac3izzfrac i i=sqrt15z$?
– gimusi
Nov 11 at 12:14












,I got $6+ frac3i=sqrt45$
– Dhamnekar Winod
Nov 11 at 12:16





,I got $6+ frac3i=sqrt45$
– Dhamnekar Winod
Nov 11 at 12:16













@DhamnekarWinod Why that? We have $bar z z=|z|^2$ and here we are ok, then $frac 3 i =-3i$ but at the RHS we should have $sqrt15z$.
– gimusi
Nov 11 at 12:21




@DhamnekarWinod Why that? We have $bar z z=|z|^2$ and here we are ok, then $frac 3 i =-3i$ but at the RHS we should have $sqrt15z$.
– gimusi
Nov 11 at 12:21












because|z|=$sqrt3$. If this is wrong,then $z=frac6-3isqrt15$
– Dhamnekar Winod
Nov 11 at 12:31





because|z|=$sqrt3$. If this is wrong,then $z=frac6-3isqrt15$
– Dhamnekar Winod
Nov 11 at 12:31












1














WLOG $z=sqrt3e^itimpliesbar z=sqrt3e^-it$ where $t$ is real



$$sqrt15=2sqrt3e^-it+dfrac3isqrt3e^it=sqrt3(2-i)e^-it$$



$$iff e^it=dfrac2-isqrt5$$



We are done.



We can go even further.



$$e^it=e^-iarcsindfrac1sqrt5$$



$$implies t=2npi -arcsindfrac1sqrt5$$ where $n$ is any integer






share|cite|improve this answer




















  • ,what is WLOG z means?
    – Dhamnekar Winod
    Nov 11 at 13:00










  • artofproblemsolving.com/wiki/…
    – lab bhattacharjee
    Nov 11 at 13:03










  • I know $e^ipi=-1$. So what is $e^it?$ and why did you multiply it by z?
    – Dhamnekar Winod
    Nov 11 at 13:11











  • math.stackexchange.com/questions/2660361/…
    – lab bhattacharjee
    Nov 11 at 13:12















1














WLOG $z=sqrt3e^itimpliesbar z=sqrt3e^-it$ where $t$ is real



$$sqrt15=2sqrt3e^-it+dfrac3isqrt3e^it=sqrt3(2-i)e^-it$$



$$iff e^it=dfrac2-isqrt5$$



We are done.



We can go even further.



$$e^it=e^-iarcsindfrac1sqrt5$$



$$implies t=2npi -arcsindfrac1sqrt5$$ where $n$ is any integer






share|cite|improve this answer




















  • ,what is WLOG z means?
    – Dhamnekar Winod
    Nov 11 at 13:00










  • artofproblemsolving.com/wiki/…
    – lab bhattacharjee
    Nov 11 at 13:03










  • I know $e^ipi=-1$. So what is $e^it?$ and why did you multiply it by z?
    – Dhamnekar Winod
    Nov 11 at 13:11











  • math.stackexchange.com/questions/2660361/…
    – lab bhattacharjee
    Nov 11 at 13:12













1












1








1






WLOG $z=sqrt3e^itimpliesbar z=sqrt3e^-it$ where $t$ is real



$$sqrt15=2sqrt3e^-it+dfrac3isqrt3e^it=sqrt3(2-i)e^-it$$



$$iff e^it=dfrac2-isqrt5$$



We are done.



We can go even further.



$$e^it=e^-iarcsindfrac1sqrt5$$



$$implies t=2npi -arcsindfrac1sqrt5$$ where $n$ is any integer






share|cite|improve this answer












WLOG $z=sqrt3e^itimpliesbar z=sqrt3e^-it$ where $t$ is real



$$sqrt15=2sqrt3e^-it+dfrac3isqrt3e^it=sqrt3(2-i)e^-it$$



$$iff e^it=dfrac2-isqrt5$$



We are done.



We can go even further.



$$e^it=e^-iarcsindfrac1sqrt5$$



$$implies t=2npi -arcsindfrac1sqrt5$$ where $n$ is any integer







share|cite|improve this answer












share|cite|improve this answer



share|cite|improve this answer










answered Nov 11 at 12:26









lab bhattacharjee

222k15156274




222k15156274











  • ,what is WLOG z means?
    – Dhamnekar Winod
    Nov 11 at 13:00










  • artofproblemsolving.com/wiki/…
    – lab bhattacharjee
    Nov 11 at 13:03










  • I know $e^ipi=-1$. So what is $e^it?$ and why did you multiply it by z?
    – Dhamnekar Winod
    Nov 11 at 13:11











  • math.stackexchange.com/questions/2660361/…
    – lab bhattacharjee
    Nov 11 at 13:12
















  • ,what is WLOG z means?
    – Dhamnekar Winod
    Nov 11 at 13:00










  • artofproblemsolving.com/wiki/…
    – lab bhattacharjee
    Nov 11 at 13:03










  • I know $e^ipi=-1$. So what is $e^it?$ and why did you multiply it by z?
    – Dhamnekar Winod
    Nov 11 at 13:11











  • math.stackexchange.com/questions/2660361/…
    – lab bhattacharjee
    Nov 11 at 13:12















,what is WLOG z means?
– Dhamnekar Winod
Nov 11 at 13:00




,what is WLOG z means?
– Dhamnekar Winod
Nov 11 at 13:00












artofproblemsolving.com/wiki/…
– lab bhattacharjee
Nov 11 at 13:03




artofproblemsolving.com/wiki/…
– lab bhattacharjee
Nov 11 at 13:03












I know $e^ipi=-1$. So what is $e^it?$ and why did you multiply it by z?
– Dhamnekar Winod
Nov 11 at 13:11





I know $e^ipi=-1$. So what is $e^it?$ and why did you multiply it by z?
– Dhamnekar Winod
Nov 11 at 13:11













math.stackexchange.com/questions/2660361/…
– lab bhattacharjee
Nov 11 at 13:12




math.stackexchange.com/questions/2660361/…
– lab bhattacharjee
Nov 11 at 13:12

















draft saved

draft discarded
















































Thanks for contributing an answer to Mathematics Stack Exchange!


  • Please be sure to answer the question. Provide details and share your research!

But avoid


  • Asking for help, clarification, or responding to other answers.

  • Making statements based on opinion; back them up with references or personal experience.

Use MathJax to format equations. MathJax reference.


To learn more, see our tips on writing great answers.





Some of your past answers have not been well-received, and you're in danger of being blocked from answering.


Please pay close attention to the following guidance:


  • Please be sure to answer the question. Provide details and share your research!

But avoid


  • Asking for help, clarification, or responding to other answers.

  • Making statements based on opinion; back them up with references or personal experience.

To learn more, see our tips on writing great answers.




draft saved


draft discarded














StackExchange.ready(
function ()
StackExchange.openid.initPostLogin('.new-post-login', 'https%3a%2f%2fmath.stackexchange.com%2fquestions%2f2993720%2fcomplex-numbers-and-conjugates%23new-answer', 'question_page');

);

Post as a guest















Required, but never shown





















































Required, but never shown














Required, but never shown












Required, but never shown







Required, but never shown

































Required, but never shown














Required, but never shown












Required, but never shown







Required, but never shown







Popular posts from this blog

Darth Vader #20

How to how show current date and time by default on contact form 7 in WordPress without taking input from user in datetimepicker

Ondo